Đến nội dung

anhquannbk nội dung

Có 474 mục bởi anhquannbk (Tìm giới hạn từ 24-05-2020)



Sắp theo                Sắp xếp  

#595476 Đề thi chọn đội tuyển dự thi VMO tỉnh Quảng Nam

Đã gửi bởi anhquannbk on 26-10-2015 - 19:37 trong Thi HSG cấp Tỉnh, Thành phố. Olympic 30-4. Đề thi và kiểm tra đội tuyển các cấp.

Đề thi chọn đội tuyển dự thi VMO tỉnh Quảng Nam 2015-2016

Hình gửi kèm

  • Snapshot_20151026_3.JPG
  • Snapshot_20151026_4.JPG



#597532 Đề thi chọn đội tuyển thi VMO của khối THPT chuyên ĐHKH Huế 2016

Đã gửi bởi anhquannbk on 09-11-2015 - 18:07 trong Thi HSG cấp Tỉnh, Thành phố. Olympic 30-4. Đề thi và kiểm tra đội tuyển các cấp.

Đề thi chọn đội tuyển thi VMO của khối THPT chuyên ĐHKH Huế 2016

Hình gửi kèm

  • 12186524_521972757972516_741439171948248741_o.jpg
  • 12185526_521972764639182_5188687048398423673_o.jpg
  • 886058_521972761305849_7785815354416487510_o.jpg



#601366 Tìm n

Đã gửi bởi anhquannbk on 03-12-2015 - 11:44 trong Tổ hợp - Xác suất và thống kê - Số phức

Tìm n 

Hình gửi kèm

  • Eqn13.gif



#602267 Tổ hợp

Đã gửi bởi anhquannbk on 08-12-2015 - 21:09 trong Tổ hợp - Xác suất và thống kê - Số phức

Có bao nhiêu số có 8 chữ số được lập thành từ các số 1,2,3,4,5,6,7,8,9, sao cho có đúng 2 số chẵn liền kề nhau




#604862 Xác suất

Đã gửi bởi anhquannbk on 23-12-2015 - 16:54 trong Tổ hợp - Xác suất và thống kê - Số phức

Năm khách hàng vào một siêu thị nhỏ gồm 5 gian hàng khác nhau. Mỗi người chọn ngẫu nhiên một gian để mua hàng. Tính xác suất để mỗi gian hàng có đúng 1 người để đến mua




#604982 Tìm m

Đã gửi bởi anhquannbk on 24-12-2015 - 08:08 trong Phương trình - hệ phương trình - bất phương trình

  Tìm m

Hình gửi kèm

  • 12434540_548204061994883_1374165180_n.jpg



#604984 Tổ hợp

Đã gửi bởi anhquannbk on 24-12-2015 - 08:47 trong Tổ hợp và rời rạc

Tổ hợp

Hình gửi kèm

  • 12415721_548211281994161_347741307_o.jpg



#604985 Topic về phương trình và hệ phương trình

Đã gửi bởi anhquannbk on 24-12-2015 - 09:07 trong Phương trình - Hệ phương trình - Bất phương trình

Đáp số: $1 <x\le 2$.

 

Hình gửi kèm

  • 12421703_548214918660464_429961097_n.jpg



#604991 Chứng minh MD, BE, AF đồng qui tại 1 điểm

Đã gửi bởi anhquannbk on 24-12-2015 - 10:14 trong Hình học

Từ M nằm ngoài (O) kẻ 2 tiếp tuyến MA, MB và cát tuyến MCD. Trên CD lấy điểm N, NA cắt (O) tại E, NB cắt (O) tại F. Chứng minh MD, BE, AF đồng qui tại 1 điểm.

Gọi K là giao điểm của AF và BE.

Áp dụng định lý Pascal cho 6 điểm trên (O) A, A, F, E, B, B ta được N, K, M thẳng hàng

 Mà M, N, D thẳng hàng

=> MD. BE, AF đồng quy




#605020 $5x^{2}+3x\sqrt{2x^{2}+1}-2=0$

Đã gửi bởi anhquannbk on 24-12-2015 - 15:08 trong Phương trình - hệ phương trình - bất phương trình

Giải PT:

$5x^{2}+3x\sqrt{2x^{2}+1}-2=0$

chuyển $3x\sqrt{2x^{2}+1}$  qua vế phải rồi bình phương là được mà




#605068 tim n de x$x^{2n}+x^{n}+1\vdots x^{2}...

Đã gửi bởi anhquannbk on 24-12-2015 - 20:19 trong Đại số

Xét n=3k, n=3k+1, n=3k+2 và sử dụng $x^{6}-1\vdots x^{2}+x+1 $, $x^{4}+x^{2}+1\vdots x^{2}+x+1 $.

n không chia hết cho 3 thỏa mãn bài toán.




#605098 C/m A = $\frac{x^{2}}{x+yz} + \f...

Đã gửi bởi anhquannbk on 24-12-2015 - 22:02 trong Bất đẳng thức và cực trị

Áp dụng bất đẳng thức Cauchy-Schwarz ta : $ \dfrac{x^{2}}{x+yz}+\dfrac{y^{2}}{y+xz}+\dfrac{z^{2}}{z+xy}$ $ \ge $ $ \dfrac{(x+y+z)^{2}}{x+y+z+xy+yz+xz} $

Theo giả thiết ta  x+y+z $ \ge $ $ \dfrac{9}{\dfrac{1}{x}+\dfrac{1}{y}+\dfrac{1}{z}} $ =9  xy+yz+xz=xyz

 Suy ra $ \dfrac{x^{2}}{x+yz}+\dfrac{y^{2}}{y+xz}+\dfrac{z^{2}}{z+xy}$ $ \ge $ $ \dfrac{9(x+y+z)}{x+y+z+xy+yz+xz} $\

 Ta cần chứng minh $ \dfrac{9}{x+y+z+xy+yz+xz} $ $ \ge $ $ \dfrac{1}{4} $
Hay x+y+z+xy+yz+xz$ \ge $ 36
Mặt khác $(xy+yz+xz)^{2}$ $ \ge $ 3xyz(x+y+z)
xy+yz+xz=xyz suy ra xy+yz+xz$ \ge $ 3(x+y+z)$ \ge $ 27
x+y+z $ \ge $ 9
Suy ra điều phải chứng minh.

Một cách khác.




#605161 CM $OE.EH=EB.ED$

Đã gửi bởi anhquannbk on 25-12-2015 - 12:47 trong Hình học

  Nhờ mọi người chứng minh một tính chất hình khá hay này

   Cho tứ giác $ABCD$ nội tiếp đường tròn $(O)$, $AC\cap BD={E}$,$AB\cap CD=F, AD\cap BC=G$, $OE\cap FG={H}$

       Chứng minh $OE.OH=EB.ED$ (hay cm $OBHD$ nội tiếp)

   Từ đó suy ra $H$ là điểm $Miken$ của tứ giác toàn phần trên.

 Giả sử gọi K là giao điểm của hai đường tròn (FBC) và (GCD)

Ta có:$ \angle GKC $+$ \angle CKF $ = $ \angle ADC $+$ \angle ABC $ =$ 180^{0}$

Suy ra G, K, F thẳng hàng

Theo tính chất phương tích ta có: 

$ GK.GF=GB.GC $=$ GO^{2} $-$ R^{2} $
$ FK.FG=FC.FD $=$ FO^{2} $-$ R^{2} $

Suy ra $ GO^{2} $-$FO^{2} $=$GK.GF-FK.FG $=$ GK^{2} $-$ FK^{2} $

Suy ra OK vuông góc với GF

Mặt khác theo định lý Brocard thì O là trực tâm tam giác EFG

nên OE vuông góc với GF tại H

Suy ra K trùng H

$ \angle DHG $=$ \angle DCG $ $\angle BHF $=$ \angle FCB $ ( do các tứ giác GHCD và FBCH nội tiếp)

suy ra $\angle DHG $=$ \angle BHF $

$\angle DHO $=$ \angle BHO $

 hay HO là phân giác góc DHB

Xét tứ giác DOHB có  HO là phân giác góc DHB mà OB=OD suy ra DOHB nội tiếp

suy ra  OE.EH=ED.EB=EA.EC

 dẫn theo OAHC nội tiếp.

 

Cách này có vẻ chặt chẽ hơn.

Hình gửi kèm

  • 12434990_549136331901656_1988705895_o.jpg



#605232 Giải phương trình $x-2\sqrt{x-1} - (x-1)\sqrt{x...

Đã gửi bởi anhquannbk on 25-12-2015 - 20:28 trong Phương trình - hệ phương trình - bất phương trình

Giải phương trình:

c/ $ \sqrt[4]{x+1} $-$ \sqrt[4]{x-1} $=$ \sqrt[4]{x} $

Đặt $ \sqrt[4]{x+1} $=a $ \sqrt[4]{x-1} $=b (a,b$ \ge 0 $)

Suy ra x=$ \dfrac{a^{4}+b^{4}}{2}$

Phương trình trở thành $ \dfrac{a^{4}+b^{4}}{2} $=$ (a-b)^{4} $ = $ (a+b)^{4} $-8ab($ a^{3}+b^{3}) $

Ta $ a^{4} $-$ b^{4} $=2

Từ đó ta hệ đối xứng




#605255 $\sqrt{x}+\sqrt{y}=10$

Đã gửi bởi anhquannbk on 25-12-2015 - 21:22 trong Đại số

1.$\left\{\begin{matrix} \sqrt{3x} +\sqrt{3y}=6& & \\ \sqrt{3x+7}+\sqrt{3y+7}=8 & & \end{matrix}\right.$

2.$\left\{\begin{matrix} \sqrt{x+1}+\sqrt{y-1}=4 & & \\ \sqrt{x+6}+\sqrt{y+4}=6 & & \end{matrix}\right.$

3.$\left\{\begin{matrix} \sqrt{x}+\sqrt{y}=10 & & \\ \sqrt{x+24}+\sqrt{y+24}=14 & & \end{matrix}\right.$

4.$\left\{\begin{matrix} (2x+3)\sqrt{4x-1}+(2y+3)\sqrt{4y-1}=2\sqrt{(2y+3)(2x+3)} & & \\ x+y=4xy & & \end{matrix}\right.$

5.$\left\{\begin{matrix} x+2y-3z=4 & & \\ \sqrt{4-x^{2}}+\sqrt{1-4y^{2}}+\sqrt{25-9z^{2}}=4\sqrt{3} & & \end{matrix}\right.$

5.$7x^{2}+7x=\sqrt{\frac{4x+9}{28}}$

7.$2x^{2}+4x=\sqrt{\frac{x+3}{2}}$

8$27x^{2}+18x=\sqrt{x+\frac{4}{3}}$.

 

Đến đây các bạn có thể giải tiếp.

Hình gửi kèm

  • 12435098_549250048556951_1729650764_o.jpg



#605310 cho $\Delta ABC$ có trực tâm $H(5;5)$, $BC: x+y...

Đã gửi bởi anhquannbk on 26-12-2015 - 10:34 trong Phương pháp tọa độ trong mặt phẳng

cho $\Delta ABC$ có trực tâm $H(5;5)$, $BC: x+y-8=0$, $M(7;3)$ và $N(4;2)$ thuộc đường tròn ngoại tiếp $\Delta ABC$. tính $S_{ABC}$

Mình nói hướng giải của mình nhé.

có phương trình đường thẳng BC suy ra được vector chỉ phương n của BC, từ đó có AH vuông góc n ta có được mối liện hệ $ x_{A} $ +$ y_{A} $ =10

Lập tọa độ  các điểm A, B, C theo các ẩn $ x_{A}, x_{B}, x_{C} $

Từ đó lập được các vec tơ AH, BC và lập phương trình vuông góc ta tính được tọa độ của A

($ x_{B} \ne x_{C} $)

Có tọa độ A và 2 điểm M, N thuộc (ABC) ta lập được phương trình  đường tròn (ABC)

Từ đó có được 2 điểm B, C

Tính được diện tích tam giác.




#605314 CM: Nếu các đường thẳng vuông góc với các cạnh BC, CA, AB lần lượt tại $...

Đã gửi bởi anhquannbk on 26-12-2015 - 10:57 trong Hình học

Bài 4. $(O)$ cắt các cạnh BC,CA,AB của tam giác ABC tại $A_1, A_2; B_1,B_2;C_1,C_2$ trong đó $A_1$ nằm giữa C và $A_2$, $B_1$ nằm giữa A và $B_2$, $C_1$ nằm giữa A và $C_2$.

CM: Nếu các đường thẳng vuông góc với các cạnh BC, CA, AB lần lượt tại $A_1,B_1,C_1$ đồng quy thì các đường thẳng vuông góc với BC,CA,AB lần lượt tại $A_2,B_2,C_2$ cũng đồng quy

__________

Mọi người giúp mình mấy bài đồng quy ở dưới nhé! Mấy bài chưa đánh dấu ở đầu nhé!

Bài 9:

Đường thẳng qua A vuông góc với EF chính là AO với O là tâm đường tròn ngoại tiếp tam giác.

tương tự 3 đường đó đồng quy tai tâm đường tròn ngoại tiếp tam giác ABC.




#605338 $\left\{\begin{matrix}\frac{1...

Đã gửi bởi anhquannbk on 26-12-2015 - 16:42 trong Đại số

Giải hệ phương trình

$\left\{\begin{matrix}\dfrac{1}{x} + \dfrac{1}{y} + \dfrac{1}{z} = 2 \\ \dfrac{2}{xy} - \dfrac{1}{z^2} = 4 \end{matrix}\right.$

Đặt $ \dfrac{1}{x}=a, \dfrac{1}{y}=b, \dfrac{1}{z}=c $

Khi đó ta :

$ a+b+c=2 $

$ 2ab-c^{2}=4 $

Rút c từ phương trình trên thế xuống dưới ta được: $ 2ab-(2-(a+b))^{2} $=4

Khai triển nhóm ta được $(a-2) ^{2}+(b-2)^{2}=0 $

suy ra $a=b=2, c=-2 $




#605369 tìm số nguyên dương a,b thỏa $\frac{a^2+b^2+ab}{ab-1...

Đã gửi bởi anhquannbk on 26-12-2015 - 19:51 trong Số học

Tìm $a,b \in \mathbb{Z}^+$ và $ab\neq 1$. Sao cho

$\frac{a^2+b^2+ab}{ab-1}\in \mathbb{Z}$

Mình thấy bài này giống một số dạng bài về sử dụng phương pháp bước nhảy Viete, bạn có thể tham khảo trên một số blog.




#605414 Xác định vị trí của A để AB.AC lớn nhất.

Đã gửi bởi anhquannbk on 26-12-2015 - 21:20 trong Hình học

1)Điểm A di động trên cung lớn BC nên $ \angle BAC $ nhọn

Ta $ S_{ABC}=\dfrac{1}{2}.AB.AC.sin\angle BAC $

Do $sin\angle BAC $ không đổi AB.AC lớn nhất khi $ S_{ABC} $ lớn nhất $ S_{ABC} $ lớn nhất khi AH lớn nhất(H hình chiếu của A trên BC)

hay A điểm chính giữa cung lớn BC




#605415 Xác định vị trí của A để AB.AC lớn nhất.

Đã gửi bởi anhquannbk on 26-12-2015 - 21:21 trong Hình học

1) Cho A là điểm di động trên cung lớn BC của (O;R). Xác định vị trí của A để AB.AC lớn nhất.

2) A là điểm cố định trên (O;R). AB, AC là 2 dây cung di động và AB.AC không đổi. Chứng minh BC luôn tiếp xúc với đường tròn cố định

1)Điểm A di động trên cung lớn BC nên $ \angle BAC $ nhọn

Ta  $ S_{ABC}=\dfrac{1}{2}.AB.AC.sin\angle BAC $

Do $sin\angle BAC $ không đổi AB.AC lớn nhất khi $ S_{ABC} $ lớn nhất $ S_{ABC} $ lớn nhất khi AH lớn nhất(H  hình chiếu của A trên BC)

hay A  điểm chính giữa cung lớn BC




#605456 $a^{2}+(b-2)^{2}> 3$ ?

Đã gửi bởi anhquannbk on 27-12-2015 - 06:54 trong Bất đẳng thức và cực trị

1/ Cho a, b, c là số đo 3 cạnh của 1 tam giác, c/m

a/ abc> (a+b-c)(b+c-a)(c+a-b) ?

b/ $a^{3}(b^{2}-c^{2})+ b^{3}(c^{2}-a^{2})+ c^{3}(a^{2}-b^{2})< 0$ với a<b<c .

2/ Cho x, y, z >0 t./m x+y+z+1=4xyz. C/m $\frac{1}{x}+\frac{1}{y}+\frac{1}{z}\geq 3$ 

3/ Cho S= $a^{2}+b^{2}+c^{2}+d^{2}+ac + bd$ trong đó ad-bc =1. C.m S $\geq \sqrt{3}$ ?

4/ C/m nếu pt $x^{4}+ax^{3}+bx^{2}+ax+1=0$ có nghiệm thì $a^{2}+(b-2)^{2}> 3$ ?

1)

$ (a+b-c)(b+c-a)\le b^{2} $

$ (a+b-c)(a+c-b)\le a^{2} $

$ (b+c-a)(a+c-b)\le c^{2} $

Nhân lại cho kết quả




#605457 $a^{2}+(b-2)^{2}> 3$ ?

Đã gửi bởi anhquannbk on 27-12-2015 - 07:25 trong Bất đẳng thức và cực trị

1/ Cho a, b, c là số đo 3 cạnh của 1 tam giác, c/m

a/ abc> (a+b-c)(b+c-a)(c+a-b) ?

b/ $a^{3}(b^{2}-c^{2})+ b^{3}(c^{2}-a^{2})+ c^{3}(a^{2}-b^{2})< 0$ với a<b<c .

2/ Cho x, y, z >0 t./m x+y+z+1=4xyz. C/m $\frac{1}{x}+\frac{1}{y}+\frac{1}{z}\geq 3$ 

3/ Cho S= $a^{2}+b^{2}+c^{2}+d^{2}+ac + bd$ trong đó ad-bc =1. C.m S $\geq \sqrt{3}$ ?

4/ C/m nếu pt $x^{4}+ax^{3}+bx^{2}+ax+1=0$ có nghiệm thì $a^{2}+(b-2)^{2}> 3$ ?

4)

$ x^{4}+ax^{3}+bx^{2}+ax+1=0 $

Chia cả 2 vế cho $ x^{2}\ne 0 $ ta được: $ (x^{2}+\dfrac{1}{x^{2}})+a(x+\dfrac{1}{x})+b+1=0 $

Đặt $ x+\dfrac{1}{x}=t $ với $ |t|\ge 2 $

phương trình trở thành $ t^{2}-2+at+b+1=0 $ $ t^{2}+at+b-1=0 $ hay $ t^{2}+1=-(at+(b-2)) $

Dùng bđt Bunhiacopxki ta được $ (t^{2}+1)^{2}=(at+(b-2))^{2}\le (t^{2}+1)(a^{2}+(b-2)^{2}) $

Suy ra $ a^{2}+(b-2)^{2}\ge\dfrac{ (t^{2}+1)^{2}}{t^{2}+1} $

Ta chỉ cần cm $ (t^{2}+1)^{2}\ge 3(t^{2}+1) $ xong dựa vào $ |t|\ge 2 $

$ t^{2}+1>3 với |t|\ge 2 $

suy ra đpcm




#605468 CM: $\frac{u_{p+1}+1}{p+1}$ là s...

Đã gửi bởi anhquannbk on 27-12-2015 - 09:38 trong Dãy số - Giới hạn

Bài này là tổng quát của bài 4 Vietnam TST 2012, các bạn tham khảo tại đây

http://diendantoanho...sach-dội-tuyển/




#605653 $\left\{\begin{matrix} x+\frac{3...

Đã gửi bởi anhquannbk on 27-12-2015 - 21:52 trong Phương trình - hệ phương trình - bất phương trình

$\left\{\begin{matrix} x+\frac{3x-y}{x^2+y^2}=3 & \\y-\frac{x+3y}{x^2+y^2}=0 & \end{matrix}\right.$

Nhân phương trình (1) với y và phương trình (2) với x sau đó cộng lại sẽ triệt tiêu phân số, rút x theo y hoặc y theo x thế vào phương trình (2)

Bài này có thể giải bằng số phức bạn tham khảo tại đây

http://diendantoanho...ệ-phương-trinh/